What is the answer to -3x-(1-x)=-15

Answers

Answer 1

Answer:

x = 7

Step-by-step explanation:

- 3x - (1 - x)= -15

-3x - 1 + x = -15

-2x - 1 = -15

-2x = -14

x = 7


Related Questions

Write an equation in slope-intercept form (y = mx + b) , that perpendicular to y = 2x - 4 that passes through the point (3, - 3) .

Answers

Answer:

Step-by-step explanation:

perp. -1/2

y + 3 = -1/2(x - 3)

y + 6/2 = -1/2x + 3/2

y = -1/2x - 3/2

what is the surface area of a rectangular prism that has a height of 47 a length of 19 and a width of 6 IF U AREN'T SYDNEETEAL I WILL REPORT U

Answers

answer:

2578

step-by-step explanation:

A=2(wl+hl+hw)=2·(47·19+6·19+6·47)=2578

good luck

hopefully this helps

have a nice day :)

Determine whether the following sequences are arithmetic. If so, identify the common
difference.
9, 15, 21, 27, ...

Answers

Answer:

Yes the sequence is arithmetic

The common difference is 6

Step-by-step explanation:

Determine whether the following sequences are arithmetic. If so, identify the common difference.

9, 15, 21, 27, ...

The sequence given in the above question is arithmetic because it follows the formula

Un = a + (n -1)d

Where

a= First term

n = Number of terms

d = Common difference

The common difference is calculated as the

Second term - First term

15 - 9

= 6

or the Third term - Second term

21 - 15

= 6

or the Fourth term - Third term

= 27 - 21

= 6

Therefore, the Common difference = 6


I NEED HELP ASAP!!!!!!!!

Answers

84!
All the sides are 18,24, and 30. Then the two triangles are 12.

Find the GCF. Show work

Answers

Answer:The greatest common factors

Step-by-step explanation:

GCF of 18 : 27 , 9            

GCF of 36 : 36                                                                                                        GCF of 27 : 9

GCF of 54 : 18

GCF of 12 : 3

GCF of 24 : 3

GCF of 40 : 40

Do not factor by grouping. Solve 2(x-2)^1/3=x+2

Answers

Step-by-step explanation:

2(x-2)⅓=x+2

so we first cube each side,such that...

(2(x-2))⅓•³=(x+2)³

we get...

2(x-2)=(x+2)³

expanding...

2x-4=x³+6x²+12x+8

grouping....

x³+6x²+12x+8-2x+4=0

solving...

x³+6x²+10x+12=0

now you can use you calculator to solve for the possible values of x

I hope this will help

What is the least common denominator of
1/7 and 2/3

Answers

Answer:

Step-by-step explanation:

The LCD here is the smallest integer evenly divisible by both 7 and 3.  That number would be 21.  Then:

1/7 and 2/3 become 3/21 and 14/21.

I need help on 12 please 14 points

Answers

Step-by-step explanation:

P = 6+6+5x-3 + 5x -3 = 10x+6

A = 6(5x-3) = 30x -18

Answer:

30x-18 is the area

the perimeter is 10x+6

Step-by-step explanation:

not enough info to solve for x, but we can still have a variable

6(5x-3)

distribute

6(5x)-6(3)

30x-18 is the area

the perimeter is 10x+6

Car survey in a survey of 1200 people who owned a certain type of car,600 said they would buy that type of car again.What percent of people surveyed were satisfied with the car?

Answers

Answer:

50 percent

Step-by-step explanation:

Total number of people in this car survey = 1200

Total number of people who they would still buy same car type = 600 (these are the satisfied people)

The percentage = 600/1200 x 100

= 0.5 x 100

= 50 percent

So the percentage of satisfied people who would buy this type of car again is 50%

What is the Mean of the data below 2,4,5,6,8,2,5, 6​

Answers

the answer is 4.75

Step-by-step explanation:

The mean of a set of numbers, sometimes simply called the average , is the sum of the data divided by the total number of data. Example 1 : ... There are 8 numbers in the set. Add them all, and then divide by 8 . so just add all you numbers and divide it by 8. simple

Select all of the ordered pairs that are solutions to the system

Answers

Answer:

ALL of them

Step-by-step explanation:

They are all inside or on the border of the graphing solution

Mary Lost 30 pounds. Before her diet she weigh 180 pounds. What is the percent of change 

Answers

Answer:16.7%

Step-by-step explanation:

150         x

_____=______

180          100

150(100)=180(x)

15000=180x

__________

180         180

x=83.3 repeated

100-83.3=16.7

Which set of three numbers can be used to make a right triangle?
Group of answer choices

4, 5, 6

3, 4, 5

5, 6, 7

Answers

Answer:

3, 4, 5

Step-by-step explanation:

[tex]a^{2} + b^{2} = c^{2}[/tex]

[tex]3^{2} + 4^{2} = 5^{2}[/tex]

9+16=25

25=25

Look at this equation. 3(x + 1) = 3x + Which number can be placed in the box that would create an equation that has an infinite number of solutions? OA) O OB) 1 C) 2 OD 3​

Answers

[tex]52 - x = 52[/tex]

[tex]x = 52 - 52[/tex]

[tex]x = 0[/tex]

Avante walked 1 and 1/2 miles in 22.5 minutes. Avante walked at a constant rate. How long did it take him to walk 1 mile?

Answers

1 1/2 miles is the same as walking 1/2 a mile 3 times, so we can divide 22.5 minutes by 3 to find how long it takes to walk half a mile
22.5/3=7.5
We can then multiply 7.5 by 2 to get how long it takes to walk 1 mile
7.5 x 2 = 15 minutes
It takes Avante 15 minutes to walk 1 mile

The value of x for the system is 1. What is the value of y?*
1 point
3x + y = 9
y = -4x + 10
1. Substitute the value of y in the first equation:
2. Combine like terms:
3. Apply the subtraction property of equality:
4. Apply the division property of equality:
3x + (-4x + 10) = 9
-x+ 10 = 9
-X = -1
x = 1
O y = 14
O y = 12
O y = 6
O y=3
When loesolved
system lisin
bstitution bet7
TA

Answers

The answer should be y=6

PLEASE HELP ME NOW!!!! DUE VERY SOON! PLEASE I BEG YOU!
Decorative glass balls are being stored in a box. Each ball has a radius of 2 1/3 cm.

How many balls will fit in each layer of a box that is 20 cm by 15 cm?


7 balls

12 balls

48 balls

Answers

Step-by-step explanation:

I'm trying to solve this problem but there's something wrong

What's the radius of one ball?

I don’t understand what it means by find the mid points someone pls help I’m so behind

Answers

Most likely they’re asking you to find the point in the middle of the lines? If yes, use the formula ((x1+x2)/2,(y1+y2)/2). Hope this helps!

Quick answer please!

Answers

12 is the answer to this equation
x (3+2) = 36 + 24

x = 12

Which characteristics best describe the undefined concept?
has length, but no width
Point
Line
Plane
2-dimensional
1-dimensional
O dimensions
location
flat surface
Intro
Done

Answers

Line. A line would have a length but has no width

A town's population went from 40,000 to 50,000. What was the percent of change in the population?

Answers

It would be a 0.8% change

Solve for x- round to the nearest tenth- Trigonometry

Answers

Answer:

sin26=11/X

0.438371146789×x=11

X. = 11/0.438371146789

X=25.09289235975

Clear selection
Using the following sequence, identify the 10th term: 2,-4, 8, -16

Answers

-1024 should be right

1. Prove that the product of the second and third numbers out of four consecutive whole numbers is two greater than the product of the first and the fourth numbers.


2. Prove that the square of the second number out of three consecutive odd numbers is four greater than the product of the first and third numbers.

Answers

Answer:

1.

(x+1)(x+2)>x(x+3)

x^2+2x+x+2>x^2+3x

x^2+3x+2>x+3x

2>0

2.

(x+2)^2>x(x+4)

x^2+4x+4>x^2+4x

4>0

Step-by-step explanation:

1.

Suppose x is the lowest number in the set of 4 consecutive numbers, then x+1, x+2, and x+3 are then the next numbers in the sequence since they are the next whole numbers.

Then, to follow the conditions of the problem, we make the equation:

(x+1)(x+2)>x(x+3)

Using the formula of (a+b)(c+d)=ac+ad+bc+bd, we can expand both sides of the equation into this:

x^2+2x+x+2>x^2+3x

=x^2+3x+2>x^2+3x

We can then cancel x^2+2x from both sides and we are left with 2>0.

2.

Again, suppose x is the lowest value in the set of consecutive numbers. In this case, odd numbers increase by 2 each time so the series of numbers are:

x, x+2, x+4

Therefore, to follow the conditions of the problem, we make the equation:

(x+2)^2>x(x+4)=

(x+2)(x+2)>x(x+4)

Using the same formula of (a+b)(c+d)=ac+ad+bc+bd, we get:

x^2+4x+4>x^2+4x

Cancel x^2+4x from both sides and we are left with 4>0

Hope this helps!

Solve the following quadratic equation using the quadratic formula.

Write the solutions in the following form, where r, s, and t are integers, and the fractions are in simplest form.

Answers

Answer:

x = (8-6i)/10

x = (8+6i)/10

Step-by-step explanation:

is it a function? and explain briefly why

Answers

Answer:

No

Step-by-step explanation:

Each output can only have one input

who uses y=mx+b in School

Answers

Everybody uses it in algebra.
people that are in algebra use it the most , they use it for finding slope and y intercept

Find the arc length of the partial circle.

Answers

Answer:

12.42 units

Step by step explanation:

Given,

length of the radius = 2 units

Therefore, arc length of the complete circle = 2 × 3.14 × 2 units = 12.56 units

Therefore, arc length of the partial circle = 3/4 × 12.56 units

= 12.42 units

is 0.24 greater than 0.42?

Answers

nope .42 is greater, .4 is closer to 1 then .2 have a great day and hope this helps

Answer:

yes it is greater

Step-by-step explanation:

PLEASE HELP! :) IMAGE IS BELOW!! QUESTION: Describe a series of transformations that will transform ∆ABC to ∆A'B'C'.

Answers

Answer:

Dilation and translation.

Hope that helps!

Step-by-step explanation:

Other Questions
In regard to exercise, which of the following statements about safety consideration is correct ? A in hot weather, exercising between 10a.m and 4 pm is Best . B exercising at 100% of your maximum heart rate is best .C exercising one hour before eating is best . D remaining well-hydrated while exercising is best . Between 30 degreesnorth and south ofthe equator, whatglobal wind belts you will find ? SOMEONEEEE HELPPPP PLEASEEE! The Mayans were one of the most significant civilizations in the Americas. Justify Researchers are studying the distribution of subscribers to a certain streaming service in different populations. From a random sample of 200 people in City C, 34 were found to subscribe to the streaming service. From a random sample of 200 people in City K, 54 were found to subscribe to the streaming service. Assuming all conditions for inference are met, which of the following is a 90 percent confidence interval for the difference in population proportions (City C minus City K) who subscribe to the streaming service?A. (0.17 0.27) + 1.65, 0.27 0.17 V 200 B. (0.17 0.27) 1.96 V (0.17)(0.83)+(0.27)(0.73) 400 C. (0.17 0.27) + 1.657 (0.17)(0.83)+(0.27)(0.73) 400 D. (0.17 0.27) + 1.96V (0.17)(0.83)+(0.27)(0.73) 200 E. (0.17 0.27) + 1.657 (0.17)(0.83)+0.27)(0.73) 200 Columbus believed he was in the islands near China.TrueFalse Will GIVE BRAINLIEST Select the correct answer.Which statement describes the end behavior of the function?12 1001 2 31 4B.A. The function approaches 0 as x approaches - and co.The function approaches 1 as x approaches -o and co.C. The function approaches 5 as x approaches -oo and co.X D. The function approaches 25 as x approaches - and co. You are studying a new gene "X" that you think controls skin color in Bearded Dragons. In order to determine what gene X does you need lots of gene X DNA to work with. So, you decide to amplify it through the process known as PCR. You have a forward and a reverse primer that have been designed to gene Xf a gene in frogs that is similar to the one found in the Bearded Dragon. You are pretty sure that these two primers will work in Bearded Dragons, so you decide to go ahead with the PCR. In frogs, the Forward Xf primer binds gene Xf between nucleotides 32 and 47. The Reverse Xf primer binds between nucleotides 1110 and 1135. What size PCR product would these two primers create if they amplified the frog gene Xfp If you had to explain the evolution of the peppered moth to a younger sibling or a cousin, how would you do it You and your friend Darren are partners for a science research project. Darren sends you a link to a website, but you don't think it's very good. List two reasons that it could possibly be not good. How did the Agricultural Revolution influence the Industrial Revolution? Ms. Scherbauer and her family go out to dinner and spend $65.20 on their meal. If they want to leave their waitress a 20% tip, how much do they pay in total? The dot plot shows the number of books Melanie's classmates have read. Approximately what percent of Melanies classmates have read more than 7 books What is the smallest integer $n$ such that $5n + 3 > -10$? ill mark brainlist plss help A program consists of 100,000 instructions as follows: Instruction Type Instruction Count Cycles per InstructionInteger arithmetic 45,000 4 Data transfer 32,000 6 Floating point arithmetic 15,000 10 Control transfer 8,000 3 Determine: a. the program execution timeb. the effective CPI for the machinec. MIPS rate for the following processors help me please help please Plz I want the answer quickly!! what is the product of the Trinomial and the Binomial below(x2+5x-2)(x-3) The next dividend payment by Wyatt, Inc., will be $2.30 per share. The dividends are anticipated to maintain a growth rate of 4.5 percent forever. If the stock currently sells for $39.85 per share, what is the required return? What is the dividend yield? What is the expected capital gains yield? Using the graph of f(x) and g(x), where g(x) = f(kx), determine the value of k. Graph of two lines. f of x passes through 2, 0 and 3, 2, and g of x passes through two thirds, 0 and 1, 2. 3 one third negative one third 3